Movatterモバイル変換


[0]ホーム

URL:


Sorry, we no longer support your browser
Please upgrade toMicrosoft Edge,Google Chrome, orFirefox. Learn more about ourbrowser support.
Skip to main content

Stack Exchange Network

Stack Exchange network consists of 183 Q&A communities includingStack Overflow, the largest, most trusted online community for developers to learn, share their knowledge, and build their careers.

Visit Stack Exchange
Loading…
Physics

Questions tagged [s-matrix-theory]

Ask Question

The S-matrix (scattering matrix) relates the initial state and the final state of a physical system undergoing a scattering process in quantum mechanics and quantum field theory. It is the unitary matrix connecting asymptotic particle states in the Hilbert space of physical states (scattering channels).

667 questions
Filter by
Sorted by
Tagged with
1vote
1answer
89views

I am working on a problem about Cutkosky’s cutting rule in Matt D. Schwartz’s Quantum Field Theory and the Standard Model. The problem asks us to show that the imaginary part of the amplitude is given ...
2votes
0answers
44views

I've been trying to understand quantum scattering, but I think I am not very clear about the full picture in the derivation of differential cross-section below. In Modern Quantum Mechanics by Sakurai, ...
1vote
0answers
94views

So by my understanding, all the particles known to exist are either spin 1, 1/2 or 0. While I don't follow the details, I've also heard the spin-statistics theorem's conclusion that in 3+1D spacetime, ...
7votes
0answers
255views

I am trying to compute the Feynman rules for a theory with derivative coupling:$$\mathscr{L}_{int} = e v^{\mu} (\partial_{\mu} \phi) \phi$$where $\phi$ is a real, massive scalar field, and $v^{\mu}...
3votes
1answer
119views

The standard Cutkosky rules go as follows:Cut the appropriate propagators (replace them with $(-2\pi i) \delta(p^2-m^2)$).Sum over all cuts.This gives $\operatorname{Im}$ (I am dropping factors ...
4votes
1answer
323views

ContextIt is well known that $S$-matrix could be computed in the following two ways:Calculate in interaction picture Perturbatively.Use the LSZ reduction.To be specific, we have the math ...
4votes
0answers
169views

Consider a general Euclidean QFT (or its lattice regularization).Given a list of all correlators of operators in this theory, and given that they are reflection-positive, how can one explicitly ...
6votes
0answers
178views

I'm reviewing the derivation of the LSZ formalism and I'm comfortable with the position-space version, which has the following form:$$\left<{p_i};{\rm out}|{q_j};{\rm in}\right> = i^{n+m}\int \...
1vote
0answers
101views

I've been trying for a while to prove how Peskin and Schroeder come up with formula 7.57 in their book (reported in the image below). But I cannot seem to understand how to derive the result. Moreover ...
3votes
0answers
95views

I am currently trying to understand the derivation of the LSZ reduction formula showed in the Book "Introduction to Quantum Field Theory" by Peskin and Schroeder. Here eq. (7.36) states:$$...
2votes
1answer
314views

I'm trying to prove the LSZ reduction formula for fermions, but I'm having some issues with signs.In particular, I found this reference that seems to use the following identities at the end of page 2:...
1vote
0answers
50views

In QM the probability is violated when:$\int \rho(\vec r,t)\vec dr\neq 1$ for a quantum mechanical system in an arbitrary state $\psi(\vec r,t)$.In this case we know that $\psi(\vec r,t)$ is the ...
3votes
0answers
129views

I have read about the Cutkosky cutting rules and optical theorem when I was studying for theoretical particle physics. I.e. Imaginary part of Greens function is directly correlated to the sum of decay ...
2votes
0answers
107views

In equation 4.87 of Peskin & Schroeder it states,$$\langle p_{1}p_{2}...|S\left|k_{A}k_{B}\right\rangle = \lim_{T\rightarrow \infty}\langle p_{1}p_{2}...|e^{-iH(2T)}\left|k_{A}k_{B}\right\rangle\...
0votes
0answers
114views

I am currently reading Weinberg The Quantum Theory of Fields, chapter $3$. Consider the following superpositions of states;\begin{equation}|\Psi_g^{\pm}(t)\rangle=\int\mathrm{d}\alpha \exp{(-iE_\...

153050per page
1
2345
45

Hot Network Questions

more hot questions
Newest s-matrix-theory questions feed

[8]ページ先頭

©2009-2025 Movatter.jp